Prove the maximum sum of products algorithm

The name of the pictureThe name of the pictureThe name of the pictureClash Royale CLAN TAG#URR8PPP











up vote
0
down vote

favorite












We are given two equal sized sets of positive integers (n integers). We can multiply any two numbers from different sets, each number being used once. All multiplied pairs are added. Prove that maximum sum pairs is when we sort the sets by highest to lowest numbers and multiply highest elements in both sets then 2nd highest and so on. I can prove for n=2 but can't seem to extend that for greater n







share|cite|improve this question















  • 1




    Often such problems yield to an attack where you use the case $n=2$ to make improvements to the arrangement of a larger number of pairs.
    – hardmath
    Jul 31 at 14:48






  • 2




    Please search for 'rearrangement inequality' on wiki.
    – J.Guldan
    Jul 31 at 14:53














up vote
0
down vote

favorite












We are given two equal sized sets of positive integers (n integers). We can multiply any two numbers from different sets, each number being used once. All multiplied pairs are added. Prove that maximum sum pairs is when we sort the sets by highest to lowest numbers and multiply highest elements in both sets then 2nd highest and so on. I can prove for n=2 but can't seem to extend that for greater n







share|cite|improve this question















  • 1




    Often such problems yield to an attack where you use the case $n=2$ to make improvements to the arrangement of a larger number of pairs.
    – hardmath
    Jul 31 at 14:48






  • 2




    Please search for 'rearrangement inequality' on wiki.
    – J.Guldan
    Jul 31 at 14:53












up vote
0
down vote

favorite









up vote
0
down vote

favorite











We are given two equal sized sets of positive integers (n integers). We can multiply any two numbers from different sets, each number being used once. All multiplied pairs are added. Prove that maximum sum pairs is when we sort the sets by highest to lowest numbers and multiply highest elements in both sets then 2nd highest and so on. I can prove for n=2 but can't seem to extend that for greater n







share|cite|improve this question











We are given two equal sized sets of positive integers (n integers). We can multiply any two numbers from different sets, each number being used once. All multiplied pairs are added. Prove that maximum sum pairs is when we sort the sets by highest to lowest numbers and multiply highest elements in both sets then 2nd highest and so on. I can prove for n=2 but can't seem to extend that for greater n









share|cite|improve this question










share|cite|improve this question




share|cite|improve this question









asked Jul 31 at 14:39









jatin

1011




1011







  • 1




    Often such problems yield to an attack where you use the case $n=2$ to make improvements to the arrangement of a larger number of pairs.
    – hardmath
    Jul 31 at 14:48






  • 2




    Please search for 'rearrangement inequality' on wiki.
    – J.Guldan
    Jul 31 at 14:53












  • 1




    Often such problems yield to an attack where you use the case $n=2$ to make improvements to the arrangement of a larger number of pairs.
    – hardmath
    Jul 31 at 14:48






  • 2




    Please search for 'rearrangement inequality' on wiki.
    – J.Guldan
    Jul 31 at 14:53







1




1




Often such problems yield to an attack where you use the case $n=2$ to make improvements to the arrangement of a larger number of pairs.
– hardmath
Jul 31 at 14:48




Often such problems yield to an attack where you use the case $n=2$ to make improvements to the arrangement of a larger number of pairs.
– hardmath
Jul 31 at 14:48




2




2




Please search for 'rearrangement inequality' on wiki.
– J.Guldan
Jul 31 at 14:53




Please search for 'rearrangement inequality' on wiki.
– J.Guldan
Jul 31 at 14:53















active

oldest

votes











Your Answer




StackExchange.ifUsing("editor", function ()
return StackExchange.using("mathjaxEditing", function ()
StackExchange.MarkdownEditor.creationCallbacks.add(function (editor, postfix)
StackExchange.mathjaxEditing.prepareWmdForMathJax(editor, postfix, [["$", "$"], ["\\(","\\)"]]);
);
);
, "mathjax-editing");

StackExchange.ready(function()
var channelOptions =
tags: "".split(" "),
id: "69"
;
initTagRenderer("".split(" "), "".split(" "), channelOptions);

StackExchange.using("externalEditor", function()
// Have to fire editor after snippets, if snippets enabled
if (StackExchange.settings.snippets.snippetsEnabled)
StackExchange.using("snippets", function()
createEditor();
);

else
createEditor();

);

function createEditor()
StackExchange.prepareEditor(
heartbeatType: 'answer',
convertImagesToLinks: true,
noModals: false,
showLowRepImageUploadWarning: true,
reputationToPostImages: 10,
bindNavPrevention: true,
postfix: "",
noCode: true, onDemand: true,
discardSelector: ".discard-answer"
,immediatelyShowMarkdownHelp:true
);



);








 

draft saved


draft discarded


















StackExchange.ready(
function ()
StackExchange.openid.initPostLogin('.new-post-login', 'https%3a%2f%2fmath.stackexchange.com%2fquestions%2f2868108%2fprove-the-maximum-sum-of-products-algorithm%23new-answer', 'question_page');

);

Post as a guest



































active

oldest

votes













active

oldest

votes









active

oldest

votes






active

oldest

votes










 

draft saved


draft discarded


























 


draft saved


draft discarded














StackExchange.ready(
function ()
StackExchange.openid.initPostLogin('.new-post-login', 'https%3a%2f%2fmath.stackexchange.com%2fquestions%2f2868108%2fprove-the-maximum-sum-of-products-algorithm%23new-answer', 'question_page');

);

Post as a guest













































































Comments

Popular posts from this blog

Relationship between determinant of matrix and determinant of adjoint?

Color the edges and diagonals of a regular polygon

What is the equation of a 3D cone with generalised tilt?